Difference between revisions of "2017 AMC 10B Problems/Problem 6"
Ishankhare (talk | contribs) |
m (→Solution) |
||
Line 6: | Line 6: | ||
==Solution== | ==Solution== | ||
− | + | We find that the volume of the larger block is <math>18</math>, and the volume of the smaller block is <math>4</math>. Dividing the two, we see that only a maximum of <math>4</math> <math>2</math> in <math>2</math> in <math>1</math> in blocks can fit inside a <math>3</math>-in by <math>2</math> in by <math>3</math> in box. <math>\qquad\textbf{(B)}\ 4</math> | |
{{AMC10 box|year=2017|ab=B|num-b=5|num-a=7}} | {{AMC10 box|year=2017|ab=B|num-b=5|num-a=7}} | ||
{{MAA Notice}} | {{MAA Notice}} |
Revision as of 11:42, 16 February 2017
Problem
What is the largest number of solid by by blocks that can fit in a by by box?
Solution
We find that the volume of the larger block is , and the volume of the smaller block is . Dividing the two, we see that only a maximum of in in in blocks can fit inside a -in by in by in box.
2017 AMC 10B (Problems • Answer Key • Resources) | ||
Preceded by Problem 5 |
Followed by Problem 7 | |
1 • 2 • 3 • 4 • 5 • 6 • 7 • 8 • 9 • 10 • 11 • 12 • 13 • 14 • 15 • 16 • 17 • 18 • 19 • 20 • 21 • 22 • 23 • 24 • 25 | ||
All AMC 10 Problems and Solutions |
The problems on this page are copyrighted by the Mathematical Association of America's American Mathematics Competitions.